You are on page 1of 77

Any emergency where response is constrained by insufficient resources Structure which is not connected to a sewerage system and is used

nd is used for
to meet immediate needs the reception, disposition and storage of feces or other excreta from the
- Extreme emergency human body
Which of the following defines a health system - Privy
- Interrelated system in which a country organizes available The main difference of a level II from a level III toilet facility
resources for the maintenance and improvement of health - Level III is connected to a sewerage system or treatment
A review of the literature prior to formulating research questions allows the plant
researcher to do which of the following? --------------------------------------------------------------------------------------------------
- AOTA The organisms that carry disease are called
According to the WHO 2013 update, non-communicable diseases are the - Vectors
leading cause of death worldwide accounting for _____ of deaths annually Protection of health by personal and communal efforts, such as enhancing
- 63% nutritional status, providing immunizations, and eliminating environmental
Crude birth rate – crude death rate = ____ risks are the actions usually done in what level of prevention
- Population growth - Primordial level of prevention
Scientific basis of electromagnetic therapy is based on The amount of a particular disease that is usually present in a community
- Separates blood cells and inc surface area thus inc O2 - Endemic
capacity Proper positioning of the baby during skin to skin contact includes
Example of discrete variable - AOTA
- Number of household members This involves conducting intensive individual interviews with a small # of
Ensuring adequate compensation for doctors who provide services respondents to explore their perspectives on a particular idea, program or
- Human resource for health situation
Time, place and person characteristics used in descriptive epidemiology - Indepth interview
are collectively termed as A way of collecting data by reviewing existing documents
- Epidemiologic variables - Document review
The family life cycle starts with this stage The main types of non-communicable diseases include
- Unattached young adult - DM, CVD, chronic respi disease, Ca
The tendency of a measurable characteristic to change from one Helping people to gain power over their own lives is defined as
individual or one setting to another - Empowerment
- Variation Measures of central tendency, except
Sheldon and leonard needs helium to conduct their experiment however - Variance
there was a shortage of helium. Their colleague, Barry, offered to give Juan, a 24 yr old resident of Quiapo just received his DSSM result. The
them his helium allocation in exchange for co-authorship in their research report says that the sputum submitted was mucopurulent with (+) for AFB
paper. Which of the following statements is true about authorship in this = +2. This means
case - 2 AFB were seen in 100 visual field
- Mere supplying of materials does not constitute authorship The final step in community diagnosis
The disease is not caused by a mosquito vector - Dissemination and evaluation of results
- Myiasis Estimating all costs and possible benefits derived from the invested
Results in under utilized coverage resource
- Insurance - Cost-benefit
What do you call the risks caused by external factors in dealing with The research participants are described in detail in which section of the
community health issues research plan
- Threats - Method
Exclusive breast feeding is done for a minimum of Which of the ff is included in the activities of the HIV/STI prevention
- 6 months program
All the procedures employed and the data gathered are perceived in the - 100% condom use program especially for entertainment
same manner by all observers establishments
- Empirical Which best describes reflexology
A method of participant selection that distorts the exposure-outcome - Pressure to the feet and hands with specific thumb, finger,
relationship from that present in the target population and hand techniques without use of needles
- Selection bias Which of the ff acts is/are prohibited in the ‘Ecological solid waste
Which of the ff is/are considered as characteristic/s of primary health care management act of 2000’?
- AOTA - AOTA are correct
The child bearing capacity of the population represented by women of Low birth weight infants
child-bearing age defined as - <2500 grams
- 15 to 49 yrs Which of the ff describes a level III water facility
A method sampling from a population when subpopulations vary - Ideal for densely populated urban areas
considerably Increase the risk of getting an inaccurate association between an
- Stratified random sampling exposure and an outcome
An individual pays a health provider or facility, from his own funds, each - If participants do not represent the population being
time he avails of medical services studied
- Out of pocket True of the exposure status of subjects in a cohort study
Areas to be improved on in a community when it comes to planning is - Exposure of interest is identified at baseline or start of the
called study
- Weaknesses The following are measure of disease frequency except
Route of admin of DTP - Relative risk
- IM 3, 6, 7, 8, 9 and 10. What is the median?
In computing perinatal mortality rate, it is important to - 7.5
- exclude still births This means that the participant’s identity, although known to the
Refers to a health facility providing the 5 key elements of the DOTS researcher, is not revealed to anyone outside of the researcher and his or
strategy her staff
- DOTS center - Confidentiality
The family assessment tool is very important in assessing the capacity of Important reminders when implement the ENC protocol
the family to participate in the provision of health care - Breathing can be stimulated by vigorous rubbing of the
- SCREEM child during towel drying
How many times can you reheat leftovers WHO recommendations for an effective malaria elimination program
- Once includes the ff
- Detection and treatment of malaria patients
Which of the following is not an ethical guideline for conducting research Number of livebirths per 1000 women between the ages of 15 to 49 is
with humans - General fertility rate
- Telling participants they must continue until the study has Comprehensive approach to prevention and control of NCDs involve
been completed which sectors/stakeholders
The global strategy on diet, physical activity and health by the WHO - AOTA
recommends the ff Top cause of under 5 mortality is
- Ensure that salt is iodized - Pneumonia
When behavior needs to be evaluated on a continuum, one can use which The set of all measurements of interest to a researcher
of the ff - Population
- Rating scale 22 cases of legionellosis occurred within 3 weeks among residents of a
When you want to do research regarding illicit drug use, you would use particular neighborhood (usually 0 or 1 per year)
what type of sampling method - Epidemic disease
- Snowball Refers to the sources which are spent in carrying out health activities so
-------------------------------------------------------------------------------------------------- far as the healthcare sector is concerned
The ff are measures of dispersion EXCEPT - Cost
- Percentile An infected person is less likely to encounter a susceptible person when a
Which of the ff should be considered when conducting an interventional large proportion of the members of the group are immune
trial - Hard immunity
- AOTA Example/s of beta blockers used in the WHO/ISH guidelines include
Key strategies employed in addressing emerging and re-emerging - Propranolol and atenolol
diseases include An individual who has contact with or who manifests the risk factor prior to
- Capability building for management, prevention and control becoming ill
of diseases that may pose an epidemic/pandemic threat - Exposed
For people to take action for themselves, the community organizer must The district hospital in Bohol has no capacity to perform an MRI to further
- Arouse, mobilize, organize diagnose a patient who is experiencing headache. The nearest tertiary
The gold standard in diagnosis of pulmonary tuberculosis is: hospital with MRI services is at Cebu. The family could not afford to travel
- Direct sputum smear microscopy to Cebu to get an MRI done. What barrier of health care utilization is most
To compute for infant mortality rate, the denominator should be evident in this case
- Number of infants born in a given year - Physical barrier
Sally relays to you her concerns about her chest pain. Her 12-lead EKG Occurs when the observed result between exposure and disease differs
shows sinus arrhythmia and a normal axis. Which of the ff would be an from the truth because of the influence of a third variable
appropriate response to Sally’s concerns to show skill in direct leading - Confounding
- Sally, I can see that you are concerned about your chest True of analytic studies in epidemiology
pain and your abnormal ECG result. Am I correct? - The purpose is to identify and quantify the relationship
The DOH program for breast feeding BF TSEK stands for between exposure and health outcomes
- Tama, Sapat at Eksklusibo Lead agency in disaster response
This involves conducting intensive individual interviews with a small - Department of social welfare and development
number of respondents to explore their perspectives on a particular idea, The risk prediction chart from the WHO utilized of cardiovascular disease
program or situation - Parameters used include gender, age and systolic blood
- Indepth interview pressure
Which is not included in maternal health care In initiation the following should be done except
- Care for menopausal women - Social investigation
A good leader looks for new leaders to train to ensure In the latest guidelines of the National Tb Control Program, the required
- Sustainability sputum samples for DSSM have been reduced to
It is a principle of doing, or permitting, no foreseeable harm including - 2
infringement of rights as a consequence of the research Which of the ff is an example of a demographic information assessing the
- Non-maleficence status of the population at a particular point in time
An ambush interview is an example of a/an - Age
- Non probability sampling DOTS stands for
The objectives of the activities during the first 30 seconds after delivery of - Directly Observed Treatment Short Course
a newborn is This refers to the harmful or hazardous use of psychoactive substances
- To stimulate breathing, provide warmth - Substance abuse
The entire group of people that is of interest Low birth weight rate is best represented as
- Population - LBW = (no of infants <25000 grams / no of all live births) x
A disposal area wherein the solid wastes are indiscriminately thrown or 100
disposed of without due planning and consideration for environmental and Numbers or figures computed from a set of data are called
health standards - Statistics
- Open dump This is a health education barrier that is needed to improve for patients to
The legislation was passed in 1976 embodying and unifying all sanitation make appropriate health decisions
laws into a single code - Health literacy
- Presidential decree no. 856 Research participants must give what before they can participate in a
Strategies to prevent Filariasis transmission include study
- AOTA - Informed consent
Example of vital statistics Identify the term that refers to a poststudy interview in which all aspects of
- AOTA the study are revealed, reasons for the use of deception are given, and
The only absolute contraindication to breastfeeding the participants’ questions are answered?
- Galactosemia - Debriefing
This is the branch of medicine that deals with the study of the causes, Which of the ff key attitudes of an effective counselor is realized by the
distribution, and control of disease in populations physician’s ability to accurately and sensitively enter into the inner private
- Epidemiology world of the patient and experience at the deepest level what the patient is
The process of deciding whose names belong on a research paper feeling
- Authorship - Empathy
Important reminders when facilitating the first breastfeeding of the This refers to the development of a comprehensive and detailed manual
newborn of procedures for data collection
- Do not throw away colostrum - Standardization of Protocol
This is a list of behaviors, characteristics or other entities that the Health education is a combination of learning experiences designed to
researcher is looking for - facilitate voluntary adaptation of behavior conductive to
- Checklist health
Epidemiology can be defined as the study of About 60 cases of gonorrhea are usually reported in this region per week,
- distribution and determinations of frequency of disease in slightly less than the national average
humans - Endemic
Which among the ff is the ancient Chinese practice of inserting needles in The law providing penalty for improper disposal of garbage and other
specific points on the body to manipulate the flow of energy or Qi forms of uncleanliness and for other purposes is directed to
- Acupuncture - All persons
Describes the course of a disease in the absence of any intervention Modes of horizontal transmission of disease, except
- Natural history of disease - Genetics
Doctors to the barrios and RNheals are programs aimed to address which The best people to identify and prioritize community health problems
of the ff - Researchers
- Human resource for health Ensuring adequate compensation for doctors who provide services
Tragic illness may result in death due to resources inadequate to health - Human resource for health
needs This is not covered under the basic mandatory immunization for all infants
- Out of pocket spending and children as it is not listed as a vaccine preventable disease
The drug of choice for mass treatment of Schistosomiasis is - Anthrax
- Diethylcarbamazine citrate + albendazole Fine organic or inorganic particles suspended in air is called
The target population of the filariasis control program includes ___ living - Particulate pollutant
in endemic municipalities in 44 provinces in 12 regions Which of the following is not included in the WHO recommendations for
- AOTA performing a screening test
Minimum treatment duration of SRDR tb is - Use a screening test that has at least a sensitivity of 85%
- 18 months and a specificity of 80% compared to a gold standard
Which describes pranic healing The extent to which a test is measuring what it is intended to measure
- Based on energy medicine - Validity
You might decide that you want to only include “boys who are in the 7th All of the following are true of the normal distribution except
grade and have been diagnosed with ADHD” in your research study. You - It has different values for the mean, median and the mode
would then, try to find 50 students who meet your ‘inclusion criteria’ and Characteristic of a stage 2 population pyramid
include them in your research study. What type of sampling would you do - More living in middle age
- Purposive The proportion of disease incidence that can be attributed to a specific
Recall is an example of what type of bias exposure
- Information bias - Attributable risk
Gram positive, spore forming, rod like bacteria is a common cause of food Felipe Makabayan went to local health officials to seek assistance about
poisoning his rodent problem at home. As a local health official, what would you
- Clostridium botulinum suggest to Mr. Makabayan?
Which of the ff represents crude mortality rate - Eliminate food sources of rodents
- CMR = (no of death cases / mid-year population) x 1000 Average annual incidence was 364 cases of pulmonary TB per 100,000
Breeding sites for anopheles mosquitoes include population in one area, compared with national average of 134 cases per
- Slow flowing shaded streams 100,000 population
The average number of yrs an individual of a given age is expected to live - Hyperendemic
if current age specific mortality rates continue to apply For any program, it is important to ____ the program after implementation
- Life- expectancy - Evaluate
Multi-sectorial coordination of the HIV/STI prevention activities is done by Which of the following is the aim of the emerging and re-emerging
which agency infectious diseases program
- Philippine National AIDS council - AOTA
The following yr, Alfredo brought Clark again for immunization. What is Refers to the difference between the value of a sample statistic, such as
the best regimen for Clark according to NIP recommendations the sample mean, and the true value of the population parameter, such as
- 1 dose (0.5ml) of influenza vaccine the population mean
Epidemiology is concerned with the frequency and pattern of health. - Sampling error
Which of the following is a measure of the risk of developing some new The quantitative and qualitative approach to health of citizens and the
condition within a specified period of time factors which influence their health is called
- Incidence - Community diagnosis
An actual threat to public safety; an exceptional event of any magnitude Level of prevention wherein measures are employed to inhibit the
that produces damage and injury demanding immediate action which may emergence of environmental, economic, social and behavioral conditions
be from natural, human generated, technological or societal origin that can cause disease
- Emergency - Primordial prevention
Factors and resources essential to the health of communities and Advantage of cohort studies
individual include - AOTA (directly measures the risk and rate of a health
- Access to nutritious foods outcome occurrence over time, requires a much smaller
This category includes diseases of long duration and generally slow study sample than case-control studies, most efficient
progression design for rare diseases
- Non-communicable diseases Which of the following transmissions occurs through contaminated
Which of the ff areas need community organizing the most materials such as food, clothes, bedding and cooking utensils
- An urban area near the estero - Indirect transmission
Based on the levels of water systems in the Philippines, piped water with Which of the ff would be included in the cause-related mortality
a communal water point, e.g. borewell, spring system is what level? - AOTA (MVA, main types of diseases, suicides or
- Level 2 homicides)
Positive political determinants of health include Are observed sets of measurements that are subsets of a corresponding
- Effective health planning population
A type of epidemiological study in which a group of people with a common - Sample
characteristic is followed over time to find how many reach a certain The effects of an intervention are measured by
health outcome of interest - Comparing the outcome in the experimental group with
- Cohort study that in a control group
The symptom that appears to consistently differentiate between PTB and The following are means of preventing NCDs associated with tobacco
non-TB respiratory disease smoking
- Chronic cough - Increasing tax and prices of tobacco products to reduce
Which is a practice in which an individual trains the mind or induces a demand
mode of consciousness This is crucial to achieve an equitable health system and setting national
- Meditation priorities in health care delivery
- Strong political will
Which of the following are controllable factors in prevention of Non- Dynamic demographic information of a population may come from which
communicable disease source
- Tobacco use - Civil registry records
Felipe Makabayan went to local health officials to seek assistance about Included in the PLISSIT model
his rodent problem at home. He found out that removing food sources was - Specific suggestions
effective and decided to eliminate breeding places at home as well such Which of the ff is a useful indicator of the level of health of the community
as - Perinatal mortality within 24 hours
- Remove excess firewood and properly dispose of old Drug of choice for falciparum malaria
furniture - Artemesin combination therapy
In declaring a shellfish ban during red tide period, the following issue/s An individual pays a premium which will cover his hospital expenses up to
must be considered a certain limit, for a period of one year
- AOTA - Insurance
Aimed at softening the impact of long-term disease and disability and The 20% discount for senior citizens on medical and dental services,
maximizes potential years of useful life diagnostic and laboratory fees, including professional fees of attending
- Tertiary prevention MDs in accordance to DOH guidelines in coordination with PhilHealth
The sustainable development goals is a universal call to action which shall be implemented in
aims to achieve which of the following - All private hospitals
- Ensure peace and prosperity for all people The technique used to identify which problems are to be addressed first
Which of the ff is true about the use of deception in research called Problem Prioritization. Which of the following is not true about
- If there is deception in a study, the participants may need problem prioritization
to be debriefed - Problems can be prioritized for short term plans only
Where should raw meat be stored in a refrigerator The consumer pays a premium which will give him comprehensive health
- At the bottom care through a ‘package of benefits’
This involves receiving an email wherein the respondents click on an - Health maintenance organizations
address to a secure website to fill in a questionnaire This measure compares standard deviations between several
- Web-based questionnaire investigations examining the same variable
This provides information about past researches related to the intended - Coefficient of variation
study. This process prevents unintentional duplication of these past Histogram is a visual representation of the frequency table. ____ is the
researches and leads into what needs to be investigated variable that we are interested in studying
- Review of related literature - X-axis
A water tight receptacle which receives the discharge of a plumbing
system or part thereof, and is designed to accomplish the partial removal
and digestion of the suspended solid matter in the sewage through a
period detention
- Septic tank
In interpreting the Odds ratio, an OR = 1 means
- The exposure does not affect the person’s odds of
developing the outcome
Which of the ff is considered as acceptable sputum sample for DSSM
- 1 spot specimen and 1 early morning specimen
These variables have distinct levels that have no inherent rank
- Nominal
Which of the following is safe to eat when raw
- Lamb
Refers to the resources which are spent in carrying out health activities so
far as the health care sector is concerned
- Cost
Milk code success is anchored upon
- AOTA
Community diagnosis include the ff
- AOTA (health status of the community, determinants of
health in the community, potential for healthy city
development
Which contraceptive would you avoid giving to a breastfeeding mother
- Combined oral contraceptives
Two or more offspring produced in the same gestation period
- Multiple
In the LGU referral system, the health center refers cases to
- District hospital
Washing of the newborn should be postponed for at least
- 3 hrs
These are activities that take place before data collection begins to
maintain integrity of the data collected
- Quality assurance
The main goal of the emerging and re-emerging infectious disease
program is/are
- Both prevention and control
The goal of the filariasis control program is to eliminate lymphatic filariasis
as a public health problem in the Philippines by year ____
- 2017
The London cholera epidemic of 1848 was traced to the Broad street
pump by whom
- Snow
Infant mortality rate is
- (no of infants diseased / no of infants under 1 year of age)
x 1000
Question 1
The period between infection and clinical onset of the disease is termed as?
Response: Incubation period
Score: 1 out of 1

Question 2
Which among the factors below in determining the nature of the infection is a measure
of the severity of disease, which can vary from very low to very high?
Response: Virulence
Score: 1 out of 1

Question 3
The test statistic that is used to compare the proportions of three different groups is
Response: Chi square test
Score: 1 out of 1

Question 4
Which of the following defines a local place variation of disease
Response: The disease has a high occurrence in both the native inhabitants and the
migrants to the place.
Score: 1 out of 1

Question 5
What does Epidemiology study?
Response: The distribution and determinants of frequency of disease in human
population
Score: 1 out of 1

Question 6
The highest score minus the lowest score equals the:
Response: Range
Score: 1 out of 1

Question 7
The London cholera epidemic of 1848 was traced to the Broad Street pump by
Response: Snow
Score: 1 out of 1

Question 8
This refers to the ability of the agent to stimulate the host to produce antibodies
Response: antigenicity
Score: 1 out of 1

Question 9
What is the appropriate measure of central tendency in qualitative data?
Response: Mode
Score: 1 out of 1

Question 10
In a frequency distribution, what is the value that divides the observed values into two
parts?
Response: Median
Score: 1 out of 1

Question 11
What is the reason why death rates are higher for males than females throughout life?
Response: All of the given reasons are true
Score: 1 out of 1

Question 12
What are changes in disease frequency usually encompassing several decades?
Response: Secular trends
Score: 1 out of 1

Question 13
Which among the following is an important source of infection maybe an infected
person who shows no evidence of clinical disease?
Response: Carrier
Score: 1 out of 1
Question 14
This is computed as the quotient of the standard deviation divided by the mean
Response: Coefficient of variation
Score: 1 out of 1

Question 15
This index estimates the risk of dying from any cause:
Response: Cause specific death rate
Score: 1 out of 1

Question 16
The term epidemic is defined as
Response: as the occurrence of cases in excess of what is normally expected in a
community or region
Score: 1 out of 1

Question 17
Predisposition and susceptibility to a disease is observed at which stage of a disease’s
natural history?
Response: Pre-pathogenesis
Score: 1 out of 1

Question 18
A measure of prognosis that identifies the number of those who died among those
diagnosed with the disease against those who were not diagnosed with the disease
Response: Relative survival
Score: 1 out of 1

Question 19
What level of prevention aims to avoid the emergence and establishment of the social,
economic and cultural patterns of living that are known to contribute to an elevated risk
of disease?
Response: Primordial Level or prevention
Score: 1 out of 1
Question 20
A trial in which systolic blood pressure was measured in the same patients during a
control period and after 2 weeks of experimental antihypertensive drug. Difference in
the average (mean) systolic blood pressure between the control and treatment periods
was measured. Which is most appropriate statistical procedure to use?
Response: Paired t test
Score: 1 out of 1

Question 21
If quarantine is necessary to contain an outbreak/epidemic, who of the following should
be quarantined?
Response: Suspects
Score: 1 out of 1

Question 22
An age dependency ratio of 105 can be interpreted as
Response: 105 persons will be supported by 100 able bodied persons
Score: 1 out of 1

Question 23
In the test of the effectiveness of a new anti-hypertensive to decrease the BP of patient,
the mean decrease was 5 mmHg with a standard deviation of 2 mmHg. Which of the
following is not true regarding the results
Response: 97% of the subjects will have either have an increase of 1mmhg in their BP
or a decrease of up to 11 mmhg
Score: 1 out of 1

Question 24
Which of the following statements regarding “herd immunity” (HI) is FALSE?
Response: HI causes the prevalence of the organism in the population to plateau, but it
will never decline
Score: 1 out of 1

Question 25
What is the type of outbreak where exposure is brief and all cases develop within one
incubation period of the disease?
Response: Point source outbreak
Score: 1 out of 1

Question 26
The following is true regarding the behavior of diseases in the community
Response: Endemic refers to the persistent occurrence of a disease in a particular
community
Score: 1 out of 1

Question 27
Susceptibility of a host to a disease can be best altered or controlled through
Response: building the immune system and resistance to agents of diseases
Score: 1 out of 1

Question 28
It is the number of existing cases of disease at a given point of time
Response: Prevalence
Score: 1 out of 1

Question 29
This refers to restriction of the activities and limitation of movements of well persons
who have been exposed to a case of communicable disease during its period of
communicability to prevent effective contact with those unexposed
Response: Quarantine
Score: 1 out of 1

Question 30
This index measures the magnitude of deaths due to a specific disease compared to
the deaths from all causes:
Response: Case fatality rate
Score: 1 out of 1

Question 31
Which of the following health indices best reflects the quality of health care delivery
system of a community?
Response: Maternal mortality rate (MMR)
Score: 1 out of 1

Question 32
The use of this rate is as a measure of overall health status for a given population
Response: Infant mortality rate
Score: 1 out of 1

Question 33
Which measure/s of central tendency is/are very sensitive to or easily affected by
extreme values?
Response: Mean
Score: 1 out of 1

Question 34
This is the measure of mortality which reveals the rate of the ten leading causes of
diseases
Response: Cause specific death rate
Score: 1 out of 1

Question 35
IMR is affected by socio demographic factors. A child will most likely not celebrate
his/her first birthday if except
Response: The child is born in an urban area
Score: 1 out of 1

Question 36
The following are description of a curve that is positively skewed
Response: Mean is greater than median
Score: 1 out of 1

Question 37
What is the graphical presentation of data most useful in locating cases during an
epidemic?
Response: Spot map
Score: 1 out of 1
Question 38
What scale of measurement is used to compare nominal parameters involving a single
population?
Response: Binomial test
Score: 1 out of 1

Question 39
When a researcher wants to test for the association between two categorical variables,
what is the appropriate statistic to use?
Response: T test
Score: 0 out of 1

Question 40
The natural history of a disease process commences with
Response: interaction of the host and disease agent factors with the environment
Score: 1 out of 1

Question 41
Pap smear is an example of what level of prevention?
Response: Secondary Level of prevention
Score: 1 out of 1

Question 42
Which of the following transmissions occurs through contaminated materials, such as
food, clothes, bedding and cooking utensils?
Response: Vehicle borne transmission
Score: 1 out of 1

Question 43
What is the characteristic of a normal distribution or Gaussian curve?
Response: mean, median and mode are equal
Score: 1 out of 1

Question 44
What scale of measurement is used to compare quantitative indicators involving two
related populations?
Response: Paired T-test
Score: 1 out of 1

Question 45
The square root of variance:
Response: Standard Deviation
Score: 1 out of 1
Answers
Here are your latest answers:

Question 1
If there are 15,231 total deaths in Country A during calendar year 2012 with a
estimated 2012 mid-year population of 2,010,787. What is the crude death rate?
Response: 757.5 deaths per 100,000 population
Score: 1 out of 1

Question 2
total number of deaths to residents in a specified geographic area (country, state,
county, etc.) divided by the total population for the
same geographic area (for a specified time period, usually a calendar year) and
multiplied by 100,000
Response: Crude death Rate
Score: 1 out of 1

Question 3
A longer life expectancy means:
Response: a higher proportion of older persons in the general population is expected in
the future
Score: 1 out of 1

Question 4
The crude death rate in Country A is 150/100,000. The crude death rate in a smaller,
developing country is 75/100,000. Based
on these data, which one of the following statements best explains the data?
Response: Crude death rates are usually higher in developed countries because of a
higher proportion of older persons in the population
Score: 1 out of 1

Question 5
If there are 119,110 total resident deaths in Country B during calendar years 2003,
2004, 2005 at 12,427,524 sum of estimated 2003, 2004, 2005 mid-year populations.
What is the crude death rate?
Response: 958.4 deaths per 100,000 population
Score: 1 out of 1

Question 6
Deaths of individuals less than 1 year of age per 1,000 live births
Response: Infant mortality rate
Score: 1 out of 1

Question 7
Variables used for the assessment of community health
Response: Health indicators
Score: 1 out of 1

Question 8
The traditional measures of health status widely used because of their ready availability
such as from death certificates which is a legal requirement in many countries
Response: Mortality Rates
Score: 1 out of 1

Question 9
The residents of Barangay Pilar were all afflicted with Acute Gastroenteritis because
mothers failed to boil the water properly resulting in the death of their children.
According to statistics, 350 children under age 5 died while 897 infants under age died.
Total population of Barangay Pilar is 3800. Compute for the Infant Mortality Rate.
Response: 0.09
Score: 1 out of 1

Question 10
In using health indicators, which statement is correct?
Response: It should be valid by actually measuring what they are supposed to measure
Score: 1 out of 1

Question 11
Rahizza, delivered in a lying-in hospital and was discharged with no complications.
However, she came back with foul-odor discharge and fever. She was subsequently
admitted and eventually died. This is the most likely cause of Rahizza's death?
Response: Infection
Score: 1 out of 1

Question 12
This is crucial to achieve an equitable health system and setting national priorities in
health care delivery
Response: Strong political will
Score: 1 out of 1

Question 13
Number of events occurring to a specified age group per 1,000 population of the
specified age group
Response: Age-specific rate
Score: 1 out of 1

Question 14
Bobby's father is a policeman. He is a good provider to his mother and two other
siblings. They are able to go to school, have their basic needs and seldom do they get
sick. However, on occasion, he would get drunk and beat up his wife and kids. This
situation most emphasizes what aspect of health according to the WHO?
Response: Health is a state of complete physical, mental and social well-being
Score: 1 out of 1

Question 15
These are factors and resources essential to the health of communities and individuals
Response: Social determinants of health
Score: 1 out of 1

Question 16
Neonatal Mortality Rate
Response: Number of individuals dying at less than 28 days of age per 1,000 live births
Score: 1 out of 1

Question 17
The overall infant mortality rate (IMR) has declined in Country A since the beginning of
the century. In some cities in 1900, up to 30% of infants
would die before reaching the age of one. Overall rates have dropped from over 800 per
100,000 to less than 10 per 100,000 in 2008.

Which of the following is the main factor responsible for the decline in IMR during the
2000s?
Response: Improvements in medical care
Score: 1 out of 1

Question 18
Fetal deaths plus neonatal deaths per 1,000 live births plus fetal deaths
Response: Perinatal mortality rate
Score: 1 out of 1

Question 19
Which health indicator reflects the equity of provision of health care
Response: Health Care Delivery Indicators
Score: 1 out of 1

Question 20
Rising life expectancy implies that
Response: more people will reach old age
Score: 1 out of 1
Question 1
Which of the following APGAR parameters refer to the satisfaction attained in solving
problems by communicating?
Response: Partnership
Score: 1 out of 1

Question 2
Engr. De Leon consulted Dr. Sotto due to persistent headache and hypertension. ON
PE, Dr. Sotto found that her blood pressure is at 110/70. In an attempt to determine
whether she is just being anxious, Dr. Sotto asked her: “You seem to be quite anxious
about your blood pressure." Which particular active listening skill is manifested by Dr.
Sotto?
Response: reflecting feeling
Score: 1 out of 1

Question 3
Which of the following is measured by the Family APGAR?
Response: Satisfaction about family relationships
Score: 1 out of 1

Question 4
A 16-year-old G1PO comes to your clinic for a routine prenatal examination. Her last
menstrual period was 5 months ago. Physical examination shows diffuse hematomas
on both upper and lower extremities, which the patient tries to hide by wearing a long-
sleeved shirt and long pants. Other Physical Examination findings were unremarkable.
Upon further inquiry, you learn that she comes from a dysfunctional family with an
abusive father and an alcoholic mother. Which of the following interventions would you
prioritize?
Response: Referral to a Women’s Health Desk section to ensure the patient’s safety at
home
Score: 1 out of 1

Question 5
Tito Vhoy refuses to talk with his doctors and even with his best friend Kristeta after
finding out he has HIV
Response: reaction to diagnosis
Score: 1 out of 1
Question 6
The family map is a tool that facilitates the communication of information about a family
system. The two parallel lines that connect two personalities indicate which of the
following relationships?
Response: functional
Score: 1 out of 1

Question 7
Which of the following is not included in the WHO recommendations for performing a
screening test?
Response: Use a screening test that has at least a sensitivity of 85% and a specificity of
80% compared to a gold standard
Score: 1 out of 1

Question 8
Dolly is a young mother of 4 adolescents. The younger children help Dolly in their sar-
sari store business when they have no schoolwork. This ability to "fill in" is classified
under which family strength?
Response: Flexibility of family functions and roles
Score: 1 out of 1

Question 9
Uses of a Family Genogram
Response: Allows the user to analyze hereditary patterns and psychological factors that
punctuate these relationships
Response: Maps out relationships and traits that may otherwise be missed on a
pedigree chart
Response: Quickly identify and understand various patterns in the patient's family
history which may have had an influence on the patient's current state of mind
Response: Graphically represens a family tree that displays detailed data about the
relationships among individuals in a family
Score: 1 out of 1

Question 10
True of a Family Genogram
Response: Must consist of 3 or more generations with each generation identified by
Roman numerals
Response: Family name is placed above each major family unit
Response: Index patient is identified with an arrow
Response: Date must be indicated when it was made to be able to adjust the ages over
time
Score: 1 out of 1

Question 11
Ruth, 39, consults you for joint and muscle pains. Physical examination and laboratory
results are normal. On probing, you note that she prefers consulting her siblings about
her personal problems. Peter, her husband, is an OFW and has not been home for the
past 2 years, but they have had more than adequate financial resources since he left.
Ruth feels that Peter is not supporting her wishes to transfer to a new neighborhood that
is closer to her parents’ house. Ruth is perennially dissatisfied with the way her children
respond to her emotional outbursts. She also complains that she hardly sees her two
teenaged kids, who are always out partying with friends. Ruth’s family can be said to be:
Response: Severely dysfunctional
Score: 1 out of 1

Question 12
4 basic situations wherein the Family APGAR is needed includes:
Response: assess a family’s capacity to participate in the provision of health care or to
cope with crisis
Score: 0 out of 1

Question 13
Which of the following changes occur when the breadwinner of the family becomes
debilitated?
Response: Second-order change
Score: 1 out of 1

Question 14
Cesar gets married and invites his new wife’s parents into his home. This family is
classified as being:
Response: Extended
Score: 1 out of 1
Question 15
As a coping mechanism, mobilizing personal control mechanisms can reduce the
impact of stress and anxiety on an individual. One personal control mechanism is
behavioral control. Which of the following corresponds to a behavioral control process
being utilized by a patient during a surgical procedure?
Response: Using breathing techniques to reduce pain during the procedure
Score: 1 out of 1

Question 16
Any loss or abnormality of psychological, physiological or anatomical structure or
function
Response: handicap
Score: 0 out of 1

Question 17
Which of the following assessment tools has the disadvantage of being limited to the
assessment of family structure?
Response: genogram
Score: 1 out of 1

Question 18
Restriction or lack (resulting from an impairment) of ability to perform an activity in the
manner or within the range considered normal
for a human being
Response: disability
Score: 1 out of 1

Question 19
In deciding which families need a more careful assessment on intimacy, the family
physician looks at which parameter?
Response: Affection
Score: 1 out of 1

Question 20
Who among the following is a candidate for hospice care?
Response: A 60-year old with stage 4 pancreatic cancer who is not in pain at the
moment

Score: 1 out of 1 . Question 1


Social investigation is done prior to any organization process in the community
Response: True
Score: 1 out of 1

Question 2
Community Organizing includes the following EXCEPT
Response: implementing a preassigned program of the LGU in the barangay
Score: 1 out of 1

Question 3
The Quantitative and qualitative approach to health of citizens and the factors which
influence their health is called
Response: Community Diagnosis
Score: 1 out of 1

Question 4
Determine if the situation described follows the community organizing principles

A coastal barangay who initiated a mangrove planting activity by themselves


Response: True
Score: 1 out of 1

Question 5
In Initiation the following should be done except
Response: social investigation
Score: 1 out of 1

Question 6
Helping people to gain power over their own lives is defined as
Response: empowerment
Score: 1 out of 1
Question 7
Community Diagnosis include the following
Response: All of the choices
Score: 1 out of 1

Question 8
The final step in community diagnosis
Response: dissemination and evaluation of results
Score: 1 out of 1

Question 9
Determine if what stage (arouse, organize, mobilize)of organizing are the following
situations:

Participating in a fun run supporting the advocacy against domestic violence


Response: Mobilize
Score: 1 out of 1

Question 10
Determine if the situation described follows the community organizing principles

A livelihood project chosen by the women of Barangay 189 choosing to make native
delicacies
Response: True
Score: 1 out of 1

Question 11
Liberating
Response: Community Based
Score: 0 out of 1

Question 12
Determine if the situation described follows the community organizing principles

A Non-Government Organization imposing its projects at barangay Pasay


Response: False
Score: 1 out of 1

Question 13
Determine the level of people's participation:

Health with the people


Response: Community Managed
Score: 0 out of 1

Question 14
Determine if the situation described follows the community organizing principles

An LGU project implemented on the grassroots level without consultation


Response: False
Score: 1 out of 1

Question 15
Aims to maintain the status quo, perpetuate the existing health system
Response: Hospital based
Score: 1 out of 1

Question 16
For people to take action for themselves, the community organizer must
Response: arouse, mobilize, organize
Score: 1 out of 1

Question 17
Determine if the situation described follows the community organizing principles :
A Medical mission organized by a non-sectarian group
Response: False
Score: 1 out of 1

Question 18
Determine if what stage (arouse, organize, mobilize)of organizing are the following
situations:

Distribution of flyers regarding cervical cancer vaccine awareness


Response: Arouse
Score: 1 out of 1

Question 19
Health is the responsibility of community health workers and leaders
Response: Community Managed
Score: 0 out of 1

Question 20
Determine if what stage (arouse, organize, mobilize)of organizing are the following
situations:

Holding a general assembly regarding the problem of waste disposal in the barangay
Response: Organize
Score: 1 out of 1
Question 1
Helping people to gain power over their own lives is defined as
Response: empowerment
Correct answer: empowerment
Score: 1 out of 1

Question 2
Which among the factors below in determining the nature of the infection is a measure
of the severity of disease, which can vary from very low to very high?
Response: Virulence
Correct answer: Virulence
Score: 1 out of 1

Question 3
Determine if what stage (arouse, organize, mobilize)of organizing are the following
situations:

Holding a general assembly regarding the problem of waste disposal in the barangay
Response: Organize
Correct answer: Organize
Score: 1 out of 1

Question 4
Determine if what stage (arouse, organize, mobilize)of organizing are the following
situations:

Participating in a fun run supporting the advocacy against domestic violence


Response: Mobilize
Correct answer: Mobilize
Score: 1 out of 1

Question 5
Which measure/s of central tendency is/are very sensitive to or easily affected by
extreme values?
Response: Mean
Correct answer: Mean
Score: 1 out of 1

Question 6
Which of the following changes occur when the breadwinner of the family becomes
debilitated?
Response: Second-order change
Correct answer: Second-order change
Score: 1 out of 1

Question 7
In deciding which families need a more careful assessment on intimacy, the family
physician looks at which parameter?
Response: Affection
Correct answer: Affection
Score: 1 out of 1

Question 8
Determine if the situation described follows the community organizing principles

A coastal barangay who initiated a mangrove planting activity by themselves


Response: True
Correct answer: True
Score: 1 out of 1

Question 9
Which of the following defines a local place variation of disease
Response: The disease has a high occurrence in both the native inhabitants and the
migrants to the place.
Correct answer: The disease has a high occurrence in both the native inhabitants and
the migrants to the place.
Score: 1 out of 1

Question 10
Which among the following is an important source of infection maybe an infected
person who shows no evidence of clinical disease?
Response: Carrier
Correct answer: Carrier
Score: 1 out of 1

Question 11
The natural history of a disease process commences with
Response: interaction of the host and disease agent factors with the environment
Correct answer: interaction of the host and disease agent factors with the environment
Score: 1 out of 1

Question 12
Primary Health Care involves health promotion, disease prevention, health
maintenance, counseling and one of the following:
Response: health education
Correct answer: health education
Score: 1 out of 1

Question 13
The traditional measures of health status widely used because of their ready availability
such as from death certificates which is a legal requirement in many countries
Response: Mortality Rates
Correct answer: Mortality Rates
Score: 1 out of 1

Question 14
In this model of people's participation in primary health care, health is the responsibility
of community health workers and leaders
Response: Community Based Model
Correct answer: Community Based Model
Score: 1 out of 1

Question 15
Restriction or lack (resulting from an impairment) of ability to perform an activity in the
manner or within the range considered normal
for a human being
Response: disability
Correct answer: disability
Score: 1 out of 1

Question 16
In Initiation the following should be done except
Response: social investigation
Correct answer: social investigation
Score: 1 out of 1

Question 17
A 16-year-old G1PO comes to your clinic for a routine prenatal examination. Her last
menstrual period was 5 months ago. Physical examination shows diffuse hematomas
on both upper and lower extremities, which the patient tries to hide by wearing a long-
sleeved shirt and long pants. Other Physical Examination findings were unremarkable.
Upon further inquiry, you learn that she comes from a dysfunctional family with an
abusive father and an alcoholic mother. Which of the following interventions would you
prioritize?
Response: Referral to a Women’s Health Desk section to ensure the patient’s safety at
home
Correct answer: Referral to a Women’s Health Desk section to ensure the patient’s
safety at home
Score: 1 out of 1

Question 18
Deaths of individuals less than 1 year of age per 1,000 live births
Response: Infant mortality rate
Correct answer: Infant mortality rate
Score: 1 out of 1

Question 19
This is computed as the quotient of the standard deviation divided by the mean
Response: Coefficient of variation
Correct answer: Coefficient of variation
Score: 1 out of 1
Question 20
Refers to what fraction of all the actual cases of disease a screening test detects
Response: Sensitivity
Correct answer: Sensitivity
Score: 1 out of 1

Question 21
Determine the level of people's participation:

Health with the people


Response: Community Based
Correct answer: Community Based
Score: 1 out of 1

Question 22
The test statistic that is used to compare the proportions of three different groups is
Response: Chi square test
Correct answer: Chi square test
Score: 1 out of 1

Question 23
Uses of a Family Genogram
Response: Allows the user to analyze hereditary patterns and psychological factors that
punctuate these relationships
Response: Maps out relationships and traits that may otherwise be missed on a
pedigree chart
Response: Quickly identify and understand various patterns in the patient's family
history which may have had an influence on the patient's current state of mind
Response: Graphically represens a family tree that displays detailed data about the
relationships among individuals in a family
Correct answer: Allows the user to analyze hereditary patterns and psychological
factors that punctuate these relationships , Maps out relationships and traits that may
otherwise be missed on a pedigree chart , Quickly identify and understand various
patterns in the patient's family history which may have had an influence on the patient's
current state of mind , Graphically represens a family tree that displays detailed data
about the relationships among individuals in a family
Score: 1 out of 1

Question 24
Delilah, delivered in a lying-in hospital and was discharged with no complications.
However, she came back with foul-odor discharge and fever. She was subsequently
admitted and eventually died. This is the most likely cause of Delilah's death?
Response: Infection
Correct answer: Infection
Score: 1 out of 1

Question 25
The family map is a tool that facilitates the communication of information about a family
system. The two parallel lines that connect two personalities indicate which of the
following relationships?
Response: functional
Correct answer: functional
Score: 1 out of 1

Question 26
Which of the following transmissions occurs through contaminated materials, such as
food, clothes, bedding and cooking utensils?
Response: Vehicle borne transmission
Correct answer: Vehicle borne transmission
Score: 1 out of 1

Question 27
Cesar gets married and invites his new wife’s parents into his home. This family is
classified as being:
Response: Extended
Correct answer: Extended
Score: 1 out of 1

Question 28
What scale of measurement is used to compare quantitative indicators involving two
related populations?
Response: Paired T-test
Correct answer: Paired T-test
Score: 1 out of 1

Question 29
Determine if the situation described follows the community organizing principles

A Non-Government Organization imposing its projects at barangay Pasay


Response: False
Correct answer: False
Score: 1 out of 1

Question 30
The Quantitative and qualitative approach to health of citizens and the factors which
influence their health is called
Response: Community Diagnosis
Correct answer: Community Diagnosis
Score: 1 out of 1

Question 31
For people to take action for themselves, the community organizer must
Response: arouse, mobilize, organize
Correct answer: arouse, mobilize, organize
Score: 1 out of 1

Question 32
As a coping mechanism, mobilizing personal control mechanisms can reduce the
impact of stress and anxiety on an individual. One personal control mechanism is
behavioral control. Which of the following corresponds to a behavioral control process
being utilized by a patient during a surgical procedure?
Response: Using breathing techniques to reduce pain during the procedure
Correct answer: Using breathing techniques to reduce pain during the procedure
Score: 1 out of 1

Question 33
Number of events occurring to a specified age group per 1,000 population of the
specified age group
Response: Age-specific rate
Correct answer: Age-specific rate
Score: 1 out of 1

Question 34
The overall infant mortality rate (IMR) has declined in Country A since the beginning of
the century. In some cities in 1900, up to 30% of infants
would die before reaching the age of one. Overall rates have dropped from over 800 per
100,000 to less than 10 per 100,000 in 2008.
Response: Improvements in medical care
Correct answer: Improvements in medical care
Score: 1 out of 1

Question 35
A trial in which systolic blood pressure was measured in the same patients during a
control period and after 2 weeks of experimental antihypertensive drug. Difference in
the average (mean) systolic blood pressure between the control and treatment periods
was measured. Which is most appropriate statistical procedure to use?
Response: Paired t test
Correct answer: Paired t test
Score: 1 out of 1

Question 36
Engr. De Leon consulted Dr. Sotto due to persistent headache and hypertension. ON
PE, Dr. Sotto found that her blood pressure is at 110/70. In an attempt to determine
whether she is just being anxious, Dr. Sotto asked her: “You seem to be quite anxious
about your blood pressure." Which particular active listening skill is manifested by Dr.
Sotto?
Response: reflecting feeling
Correct answer: reflecting feeling
Score: 1 out of 1

Question 37
Any loss or abnormality of psychological, physiological or anatomical structure or
function
Response: handicap
Correct answer: impairment
Score: 0 out of 1

Question 38
What is the appropriate measure of central tendency in qualitative data?
Response: Mode
Correct answer: Mode
Score: 1 out of 1

Question 39
Determine if the situation described follows the community organizing principles

A livelihood project chosen by the women of Barangay 189 choosing to make native
delicacies
Response: True
Correct answer: True
Score: 1 out of 1

Question 40
The final step in community diagnosis
Response: dissemination and evaluation of results
Correct answer: dissemination and evaluation of results
Score: 1 out of 1

Question 41
Dolly is a young mother of 4 adolescents. The younger children help Dolly in their sar-
sari store business when they have no schoolwork. This ability to "fill in" is classified
under which family strength?
Response: Flexibility of family functions and roles
Correct answer: Flexibility of family functions and roles
Score: 1 out of 1

Question 42
Benefit of primary health care when a primary care physician who is well acquainted
with a patient provides more personal and humane medical care and does so more
economically than a physician involved only in episodic care.
Response: Cost Effective Care
Correct answer: Cost Effective Care
Score: 1 out of 1

Question 43
Toteng's father is a fisherman. He is a good provider to his mother and two other
siblings. They are able to go to school, have their basic needs and seldom do they get
sick. However, on occasion, he would get drunk and beat up his wife and kids. This
situation most emphasizes what aspect of health according to the WHO?
Response: Health is a state of complete physical, mental and social well-being
Correct answer: Health is a state of complete physical, mental and social well-being
Score: 1 out of 1

Question 44
The London cholera epidemic of 1848 was traced to the Broad Street pump by
Response: Snow
Correct answer: Snow
Score: 1 out of 1

Question 45
True of a Family Genogram
Response: Must consist of 3 or more generations with each generation identified by
Roman numerals
Response: Family name is placed above each major family unit
Response: Index patient is identified with an arrow
Response: Date must be indicated when it was made to be able to adjust the ages over
time
Correct answer: Must consist of 3 or more generations with each generation identified
by Roman numerals , Family name is placed above each major family unit , Index
patient is identified with an arrow , Date must be indicated when it was made to be able
to adjust the ages over time
Score: 1 out of 1

Question 46
Community Organizing includes the following EXCEPT
Response: implementing a preassigned program of the LGU in the barangay
Correct answer: implementing a preassigned program of the LGU in the barangay
Score: 1 out of 1

Question 47
In using health indicators, which statement is correct?
Response: It should be valid by actually measuring what they are supposed to measure
Correct answer: It should be valid by actually measuring what they are supposed to
measure
Score: 1 out of 1

Question 48
This index estimates the risk of dying from any cause:
Response: Case-fatality rate
Correct answer: Cause specific death rate
Score: 0 out of 1

Question 49
Determine if the situation described follows the community organizing principles :
A Medical mission organized by a non-sectarian group
Response: False
Correct answer: False
Score: 1 out of 1

Question 50
An age dependency ratio of 105 can be interpreted as
Response: 105 persons will be supported by 100 able bodied persons
Correct answer: 105 persons will be supported by 100 able bodied persons
Score: 1 out of 1

Question 51
Determine if what stage (arouse, organize, mobilize)of organizing are the following
situations:

Distribution of flyers regarding cervical cancer vaccine awareness


Response: Arouse
Correct answer: Arouse
Score: 1 out of 1

Question 52
total number of deaths to residents in a specified geographic area (country, state,
county, etc.) divided by the total population for the
same geographic area (for a specified time period, usually a calendar year) and
multiplied by 100,000
Response: Case Specific Death rate
Correct answer: Crude death Rate
Score: 0 out of 1

Question 53
According to the primary care referral system in the Philippines, which corresponds to
the proper heirarchy of referral?
Response: Health Center to District Hospital
Correct answer: Health Center to District Hospital
Score: 1 out of 1

Question 54
The square root of variance:
Response: Standard Deviation
Correct answer: Standard Deviation
Score: 1 out of 1

Question 55
Implementation of the Seat belt Law and Helmet Law is a form of:
Response: Primordial Level or prevention
Correct answer: Primordial Level or prevention
Score: 1 out of 1

Question 56
In a frequency distribution, what is the value that divides the observed values into two
parts?
Response: Median
Correct answer: Median
Score: 1 out of 1

Question 57
Will, a 24 year old blacksmith, has been experiencing right shoulder pain for the past
10 days but continued to work on his projects. However, increasing pain on the affected
area is preventing her from finishing her work. In this case she tends to perceive her
health as a measure of:
Response: Satisfactory functioning
Correct answer: Satisfactory functioning
Score: 1 out of 1

Question 58
Which of the following APGAR parameters refer to the satisfaction attained in solving
problems by communicating?
Response: Partnership
Correct answer: Partnership
Score: 1 out of 1

Question 59
Activities falling under Primary Prevention include the :
Response: Activities falling under Primary Prevention include the :
Correct answer: Activities falling under Primary Prevention include the :
Score: 1 out of 1

Question 60
Which of the following is measured by the Family APGAR?
Response: Satisfaction about family relationships
Correct answer: Satisfaction about family relationships
Score: 1 out of 1

Question 61
The crude death rate in Country A is 150/100,000. The crude death rate in a smaller,
developing country is 75/100,000. Based
on these data, which one of the following statements best explains the data?
Response: Crude death rates are usually higher in developed countries because of a
higher proportion of older persons in the population
Correct answer: Crude death rates are usually higher in developed countries because of
a higher proportion of older persons in the population
Score: 1 out of 1

Question 62
Pap smear is an example of what level of prevention?
Response: Secondary Level of prevention
Correct answer: Secondary Level of prevention
Score: 1 out of 1

Question 63
Rising life expectancy implies that
Response: more people will reach old age
Correct answer: more people will reach old age
Score: 1 out of 1

Question 64
Stage wherein the disease process is underway but no symptoms of disease have
become apparent
Response: Latent (Hidden) Stage
Correct answer: Latent (Hidden) Stage
Score: 1 out of 1

Question 65
This is the measure of mortality which reveals the rate of the ten leading causes of
diseases
Response: Cause specific death rate
Correct answer: Cause specific death rate
Score: 1 out of 1

Question 66
Which of the following assessment tools has the disadvantage of being limited to the
assessment of family structure?
Response: genogram
Correct answer: genogram
Score: 1 out of 1

Question 67
If quarantine is necessary to contain an outbreak/epidemic, who of the following should
be quarantined?
Response: Suspects
Correct answer: Suspects
Score: 1 out of 1

Question 68
Essential health care based on practical, scientifically sound and socially acceptable
methods and technology made universally accessible to individuals
and families in the community through their full participation and at a cost that the
community and country can afford to maintain at every stage of their development in the
spirit of self-reliance and self-determination
Response: Primary Health Care
Correct answer: Primary Health Care
Score: 1 out of 1

Question 69
Predisposition and susceptibility to a disease is observed at which stage of a disease’s
natural history?
Response: Pre-pathogenesis
Correct answer: Pre-pathogenesis
Score: 1 out of 1

Question 70
Who among the following is a candidate for hospice care?
Response: A 60-year old with stage 4 pancreatic cancer who is not in pain at the
moment
Correct answer: A 60-year old with stage 4 pancreatic cancer who is not in pain at the
moment
Score: 1 out of 1

Question 71
Disease initiation or progression depends on the interaction of the following:
Response: Host, Environment and Agent
Correct answer: Host, Environment and Agent
Score: 1 out of 1

Question 72
Results of test when individuals are mistakenly classified as sick when they are actually
healthy
Response: Sensitivity
Correct answer: False Positive
Score: 0 out of 1

Question 73
The term epidemic is defined as
Response: as the occurrence of cases in excess of what is normally expected in a
community or region
Correct answer: as the occurrence of cases in excess of what is normally expected in a
community or region
Score: 1 out of 1

Question 74
According to the WHO, health is
Response: A state of complete physical, mental, and social well-being and not merely
the absence of disease or infirmity
Correct answer: A state of complete physical, mental, and social well-being and not
merely the absence of disease or infirmity
Score: 1 out of 1

Question 75
This is crucial to achieve an equitable health system and setting national priorities in
health care delivery
Response: Strong political will
Correct answer: Strong political will
Score: 1 out of 1

Question 76
If there are 15,231 total deaths in Country A during calendar year 2012 with a
estimated 2012 mid-year population of 2,010,787. What is the crude death rate?
Response: 757.5 deaths per 100,000 population
Correct answer: 757.5 deaths per 100,000 population
Score: 1 out of 1

Question 77
Ruth, 39, consults you for joint and muscle pains. Physical examination and laboratory
results are normal. On probing, you note that she prefers consulting her siblings about
her personal problems. Peter, her husband, is an OFW and has not been home for the
past 2 years, but they have had more than adequate financial resources since he left.
Ruth feels that Peter is not supporting her wishes to transfer to a new neighborhood that
is closer to her parents’ house. Ruth is perennially dissatisfied with the way her children
respond to her emotional outbursts. She also complains that she hardly sees her two
teenaged kids, who are always out partying with friends. Ruth’s family can be said to be:
Response: Severely dysfunctional
Correct answer: Severely dysfunctional
Score: 1 out of 1

Question 78
This index measures the magnitude of deaths due to a specific disease compared to
the deaths from all causes:
Response: Case fatality rate
Correct answer: Case fatality rate
Score: 1 out of 1

Question 79
What is the reason why death rates are higher for males than females throughout life?
Response: All of the given reasons are true
Correct answer: All of the given reasons are true
Score: 1 out of 1

Question 80
These are factors and resources essential to the health of communities and individuals
Response: Social determinants of health
Correct answer: Social determinants of health
Score: 1 out of 1

Question 81
What is the type of outbreak where exposure is brief and all cases develop within one
incubation period of the disease?
Response: Point source outbreak
Correct answer: Point source outbreak
Score: 1 out of 1

Question 82
Liberating
Response: Community Managed
Correct answer: Community Managed
Score: 1 out of 1

Question 83
What is the graphical presentation of data most useful in locating cases during an
epidemic?
Response: Spot map
Correct answer: Spot map
Score: 1 out of 1

Question 84
Susceptibility of a host to a disease can be best altered or controlled through
Response: building the immune system and resistance to agents of diseases
Correct answer: building the immune system and resistance to agents of diseases
Score: 1 out of 1

Question 85
Social investigation is done prior to any organization process in the community
Response: True
Correct answer: True
Score: 1 out of 1

Question 86
The residents of Barangay Pilar were all afflicted with Acute Gastroenteritis because
mothers failed to boil the water properly resulting in the death of their children.
According to statistics, 350 children under age 5 died while 897 infants under age died.
Total population of Barangay Pilar is 3800. Compute for the Infant Mortality Rate.
Response: 0.09
Correct answer: 0.09
Score: 1 out of 1

Question 87
The following is true regarding the behavior of diseases in the community
Response: Endemic refers to the persistent occurrence of a disease in a particular
community
Correct answer: Endemic refers to the persistent occurrence of a disease in a particular
community
Score: 1 out of 1

Question 88
What is the characteristic of a normal distribution or Gaussian curve?
Response: mean, median and mode are equal
Correct answer: mean, median and mode are equal
Score: 1 out of 1

Question 89
Aims to maintain the status quo, perpetuate the existing health system
Response: Hospital based
Correct answer: Hospital based
Score: 1 out of 1

Question 90
Community Diagnosis include the following
Response: All of the choices
Correct answer: All of the choices
Score: 1 out of 1

Question 91
4 basic situations wherein the Family APGAR is needed includes:
Response: When the family is directly involved in caring for the patient
Response: When treating a new patient in order to get information to serve as general
view of family function
Response: When treating a patient whose family is in crisis
Response: When a patient’s behavior makes you suspect a psychosocial problem
possibly due to family dysfunction
Response: assess a family’s capacity to participate in the provision of health care or to
cope with crisis
Correct answer: When the family is directly involved in caring for the patient , When
treating a new patient in order to get information to serve as general view of family
function , When treating a patient whose family is in crisis , When a patient’s behavior
makes you suspect a psychosocial problem possibly due to family dysfunction
Score: 0 out of 1

Question 92
Which of the following statements regarding “herd immunity” (HI) is FALSE?
Response: HI causes the prevalence of the organism in the population to plateau, but it
will never decline
Correct answer: HI causes the prevalence of the organism in the population to plateau,
but it will never decline
Score: 1 out of 1

Question 93
Tito Vhoy refuses to talk with his doctors and even with his best friend Kristeta after
finding out he has HIV
Response: reaction to diagnosis
Correct answer: reaction to diagnosis
Score: 1 out of 1

Question 94
The normal course of a disease in the absence of intervention
Response: Natural History of Disease
Correct answer: Natural History of Disease
Score: 1 out of 1
Question 95
Neonatal Mortality Rate
Response: Number of individuals dying at less than 28 days of age per 1,000 live births
Correct answer: Number of individuals dying at less than 28 days of age per 1,000 live
births
Score: 1 out of 1

Question 96
Refers to whether the test identifies only those with the disease, or does it mistakenly
classify some healthy people as being sick
Response: Sensitivity
Correct answer: Specificity
Score: 0 out of 1

Question 97
Determine if the situation described follows the community organizing principles

An LGU project implemented on the grassroots level without consultation


Response: False
Correct answer: False
Score: 1 out of 1

Question 98
Which of the following is not included in the WHO recommendations for performing a
screening test?
Response: Use a screening test that has at least a sensitivity of 85% and a specificity of
80% compared to a gold standard
Correct answer: Use a screening test that has at least a sensitivity of 85% and a
specificity of 80% compared to a gold standard
Score: 1 out of 1

Question 99
Health is the responsibility of community health workers and leaders
Response: Community Based
Correct answer: Community Based
Score: 1 out of 1

Question 100
The use of this rate is as a measure of overall health status for a given population
Response: Infant mortality rate
Correct answer: Infant mortality rate
Score: 1 out of 1
RESEARCH
Question 1
A randomized clinical trial is undertaken to examine the effect of a new combination of
antiretroviral drugs on HIV viral load compared to usual therapy. Randomization is used
for allocation of subjects to either treatment or control (usual care) groups in
experimental studies.
Randomization ensures that:
Response: Placebo effects are eliminated
Correct answer: Assignment occurs by chance
Score: 0 out of 1

Question 2
The endpoint or “tool” against which a certain intervention or exposure is measured
Response: Outcome
Correct answer: Outcome
Score: 1 out of 1

Question 3
It is the probability of the person not having the disease when the test is negative.
Response: negative predictive value
Correct answer: negative predictive value
Score: 1 out of 1

Question 4
Is a class of research methods that involve observation of all of a population, or a
representative subset, at one specific point in time.
Response: case control
Correct answer: cross sectional
Score: 0 out of 1

Question 5
The description in 1981 of four young men with a previously rare form of pneumonia
was the first in a wide range of epidemiological studies on the condition that became
known as the acquired immunodeficiency syndrome (AIDS) is an example of what type
of study.
Response: case series
Correct answer: case series
Score: 1 out of 1

Question 6
It is the proportion of people with the disease in the screened population who are
identified as ill by the screening test.
Response: sensitivity
Correct answer: sensitivity
Score: 1 out of 1

Question 7
The following 2 × 2 table represents the findings of a five-year cohort study in which
the incidence of suicide in veterans who served in Vietnam was compared with that
of veterans who served elsewhere.

The relative risk is computed as


Response: [a/(a+b)]-[c/(c+d)]
Correct answer: [a/(a + b)]/[c/(c + d)]
Score: 0 out of 1

Question 8
What is the study design where the most accurate relative risk is established?
Response: cross sectional
Correct answer: cohort concurrent
Score: 0 out of 1

Question 9
What does Double blind determination of the exposure and outcome do?
Response: avoiding observer and subject bias
Correct answer: avoiding observer and subject bias
Score: 1 out of 1

Question 10
In a cohort study of the association of leukemia with x-ray exposure, the relative risk
was 3. What does this mean?
Response: X-ray exposure protects the person from developing leukemia
Correct answer: Those with x-ray exposure have threefold probability of developing
leukemia
Score: 0 out of 1

Question 11
Refers to the alternative intervention or exposure
Response: Intervention
Correct answer: Comparison
Score: 0 out of 1

Question 12
It is the principles of doing, or permitting, no foreseeable harm including infringement of
rights as a consequence of the research.
Response: respect for persons
Correct answer: non-maleficence
Score: 0 out of 1

Question 13
In a study of the cause of lung cancer, patients who had the disease were matched
with controls by age, sex, place of residence, and social class. The frequency of
cigarette smoking was then compared in the two groups. What type of study was this?
Response: retrospective cohort
Correct answer: case control
Score: 0 out of 1

Question 14
It involves recognition of the personal dignity and autonomy of individuals, and special
protection of those persons with diminished autonomy.
Response: respect for persons
Correct answer: respect for persons
Score: 1 out of 1

Question 15
What is the study design comparing the prevalence of hepatitis B infection among
government and private hospitals?
Response: case-control
Correct answer: cross sectional
Score: 0 out of 1

Question 16
The association between low birth weight and maternal smoking during pregnancy can
be studied by obtaining smoking histories from women at the time of the first prenatal
visit and then subsequently assessing and assigning birth weight at delivery according
to smoking histories. What type of study is this?
Response: prospective cohort
Correct answer: prospective cohort
Score: 1 out of 1

Question 17
An investigation aimed at ascertaining the status of a set of variables, such as the
number and variety of persons with specific conditions in a specified population, but
without any critical analysis or attempt to test casual hypotheses.
Response: analytical study
Correct answer: descriptive study
Score: 0 out of 1

Question 18
What type of study has this objective: To determine whether maternal deficiency of
folate is a cause of congenital defects of the neural tube? Mothers of 100 newborns with
congenital neural tube defects and 200 newborns without congenital neural tube defects
were questioned about intake of multivitamins and folate during pregnancy.
Response: cross sectional
Correct answer: case-control
Score: 0 out of 1

Question 19
It is an epidemiological experiment designed to study the effects of a particular
intervention, usually a treatment for a specific disease.
Response: cross sectional
Correct answer: randomized control trial
Score: 0 out of 1

Question 20
The following 2 × 2 table represents the findings of a five-year cohort study in which
the incidence of suicide in veterans who served in Vietnam was compared with that
of veterans who served elsewhere.

The odds ratio is computed as


Response: ad/bc
Correct answer: ad/bc
Score: 1 out of 1\

HEALTH LAWS AND PROGRAMS

Question 1
Primary unit with the responsibility of reporting the Notifiable diseases
Response: Disease Reporting Unit
Correct answer: Disease Reporting Unit
Score: 1 out of 1

Question 2
the addition of nutrients to processed foods at levels above the natural state
Response: Salt iodization
Correct answer: Food fortification
Score: 0 out of 1

Question 3
A senior citizen is...
Response: any resident citizen of the Philippines at least 60 years old
Correct answer: any resident citizen of the Philippines at least 60 years old
Score: 1 out of 1

Question 4
Finished, labelled, medicinal products that contain as active ingredient/s serial or
underground part/s of plant or other materials or combination thereof, whether in the
crude state or as plant preparations
Response: Natural product
Correct answer: Herbal medicines
Score: 0 out of 1

Question 5
Included in the Immediately Notifiable Diseases list
Response: All of the choices are correct
Correct answer: Rabies
Score: 0 out of 1

Question 6
Autopsies may be performed on patients who die in accredited hospitals subject to the
following requirements
Response: All choices are correct
Correct answer: All choices are correct
Score: 1 out of 1

Question 7
This type of nuisance is NOT included in the Code of Sanitation
Response: Politicians who grandstand during senate hearings
Correct answer: Politicians who grandstand during senate hearings
Score: 1 out of 1

Question 8
This person is NOT authorized under the Code of Sanitation to perform autopsy and
dissection of remains
Response: Health officers
Correct answer: Any Licensed physician
Score: 0 out of 1

Question 9
The sum total of knowledge, skills, and practice on health care, not necessarily
explicable in the context of modern, scientific philosophical framework, but recognized
by the people to help maintain and improve their health towards the wholeness of their
being, the community and society, and their interrelations based on culture, history,
heritage, and consciousness
Response: Alternative health care modalities
Correct answer: Traditional Medicine
Score: 0 out of 1

Question 10
The particular aim of ASIN (An Act for Salt Iodization Nationwide) is to eliminate
Response: Iodine deficiency
Correct answer: Iodine deficiency
Score: 1 out of 1

Question 11
The following shall use generic terminology according to the Generics Act
Response: Veterinarians
Correct answer: All of the choices are correct
Score: 0 out of 1

Question 12
a disorder resulting from deficiencies in Vitamin A, iron, iodine and other micronutrients
which the body needs in minute quantities everyday
Response: Iodine deficiency disorders
Correct answer: Micronutrient malnutrition
Score: 0 out of 1

Question 13
Which is considered a form of biomedicine?
Response: All choices are correct
Correct answer: All choices are correct
Score: 1 out of 1

Question 14
Which are classified as Alternative health care modalities?
Response: Chiropractics
Correct answer: Chiropractics
Score: 1 out of 1

Question 15
The Rabies Program is jointly implemented by the DOH with which of the following
government agencies?
Response: Public Information Office
Correct answer: Department of Agriculture
Score: 0 out of 1

Question 16
Medicines access program designed to reach the poorest of the poor with complete
treatment regimens for the top most common diseases in the country
Response: Generics Act
Correct answer: Complete Treatment Pack Program (ComPack)
Score: 0 out of 1

Question 17
Included in the Weekly Notifiable Diseases
Response: Acute flaccid paralysis
Correct answer: Acute bloody diarrhea
Score: 0 out of 1

Question 18
The project is developed as an early warning system designed to monitor diseases
(both communicable and non-communicable), and health trends, that can be harnessed
as a powerful tool by health emergency managers in getting vital information for
appropriate and timely response during emergencies and disasters
Response: Philippine Infectious Disease Surveillance and Reporting (PIDSR)
Correct answer: Surveillance in Post-Extreme Emergencies and Disasters (SPEED)
Score: 0 out of 1

Question 19
The National Dengue Control Program is directed towards what strategy in dengue
prevention and control in endemic areas?
Response: Individual Approach
Correct answer: Community Based Approach
Score: 0 out of 1

Question 20
To be considered as a benefactor of a senior citizen under the Senior Citizen's act
Response: related or not to the senior citizen who takes care of him/her as a dependent
Correct answer: related or not to the senior citizen who takes care of him/her as a
dependent
Score: 1 out of 1

Question 21
When there are no notifiable cases for a required reporting period, the Disease
reporting unit must...
Response: not necessarily submit any report
Correct answer: submit "zero case" reporting
Score: 0 out of 1
RUN THRU

Question 1
Who is considered a senior citizen to avail of benefits and privileges under Philippine
law?
Response: Any resident citizen of the Philippines at least sixty (60) years old
Correct answer: Any resident citizen of the Philippines at least sixty (60) years old
Score: 1 out of 1

Question 2
This type of nuisance is NOT included in the Code of Sanitation
Response: Politicians who tolerate the above
Correct answer: Politicians who tolerate the above
Score: 1 out of 1

Question 3
The term epidemic is defined as
Response: as the occurrence of cases in excess of what is normally expected in a
community or region
Correct answer: as the occurrence of cases in excess of what is normally expected in a
community or region
Score: 1 out of 1

Question 4
What does Epidemiology study?
Response: The distribution and determinants of frequency of disease in human
population
Correct answer: The distribution and determinants of frequency of disease in human
population
Score: 1 out of 1

Question 5
True of the Stages of Change model
Response: The stages of change are called precontemplation, contemplation,
preparation, action, and response
Correct answer: Change is a process, and patients have different counseling and
informational needs depending on where they are in this process
Score: 0 out of 1

Question 6
Theory on behavior change wherein the focus in on personal factors, environmental
factors, and human behavior exert influence on each other
Response: Social cognitive theory
Correct answer: Social cognitive theory
Score: 1 out of 1

Question 7
As a coping mechanism, mobilizing personal control mechanisms can reduce the
impact of stress and anxiety on an individual. One personal control mechanism is
behavioral control. Which of the following corresponds to a behavioral control process
being utilized by a patient during a surgical procedure?
Response: Using breathing techniques to reduce pain during the procedure
Correct answer: Using breathing techniques to reduce pain during the procedure
Score: 1 out of 1

Question 8
Determine if what stage (arouse, organize, mobilize)of organizing are the following
situations:

Distribution of flyers regarding cervical cancer vaccine awareness


Response: Arouse
Correct answer: Arouse
Score: 1 out of 1

Question 9
The following shall use generic terminology according to the Generics Act:
Response: Government physicians
Correct answer: All of the choices are correct
Score: 0 out of 1

Question 10
Which of the following APGAR parameters refer to the satisfaction attained in solving
problems by communicating?
Response: Partnership
Correct answer: Partnership
Score: 1 out of 1

Question 11
Community Organizing includes the following EXCEPT
Response: implementing a preassigned program of the LGU in the barangay
Correct answer: implementing a preassigned program of the LGU in the barangay
Score: 1 out of 1

Question 12
The test statistic that is used to compare the proportions of three different groups is
Response: Chi square test
Correct answer: Chi square test
Score: 1 out of 1

Question 13
4 basic situations wherein the Family APGAR is needed includes:
Response: When the family is directly involved in caring for the patient
Response: When treating a new patient in order to get information to serve as general
view of family function
Response: When treating a patient whose family is in crisis
Response: When a patient’s behavior makes you suspect a psychosocial problem
possibly due to family dysfunction
Correct answer: When the family is directly involved in caring for the patient , When
treating a new patient in order to get information to serve as general view of family
function , When treating a patient whose family is in crisis , When a patient’s behavior
makes you suspect a psychosocial problem possibly due to family dysfunction
Score: 1 out of 1

Question 14
If there are 119,110 total resident deaths in Country B during calendar years 2003,
2004, 2005 at 12,427,524 sum of estimated 2003, 2004, 2005 mid-year populations.
What is the crude death rate?
Response: 958.4 deaths per 100,000 population
Correct answer: 958.4 deaths per 100,000 population
Score: 1 out of 1

Question 15
Which of the following transmissions occurs through contaminated materials, such as
food, clothes, bedding and cooking utensils?
Response: Vehicle borne transmission
Correct answer: Vehicle borne transmission
Score: 1 out of 1

Question 16
Bobby's father is a policeman. He is a good provider to his mother and two other
siblings. They are able to go to school, have their basic needs and seldom do they get
sick. However, on occasion, he would get drunk and beat up his wife and kids. This
situation most emphasizes what aspect of health according to the WHO?
Response: Health is a state of complete physical, mental and social well-being
Correct answer: Health is a state of complete physical, mental and social well-being
Score: 1 out of 1

Question 17
total number of deaths to residents in a specified geographic area (country, state,
county, etc.) divided by the total population for the
same geographic area (for a specified time period, usually a calendar year) and
multiplied by 100,000
Response: Crude death Rate
Correct answer: Crude death Rate
Score: 1 out of 1

Question 18
Deaths of individuals less than 1 year of age per 1,000 live births
Response: Infant mortality rate
Correct answer: Infant mortality rate
Score: 1 out of 1

Question 19
Health is the responsibility of community health workers and leaders
Response: Community Managed
Correct answer: Community Based
Score: 0 out of 1

Question 20
Epidemiology can be defined as the study of
Response: The distribution and determinations of frequency of disease in humans
Correct answer: The distribution and determinations of frequency of disease in humans
Score: 1 out of 1

Question 21
Theory on behavior change wherein the focus is on the Individuals' perceptions of the
threat posed by a health problem, the benefit of avoiding the threat, and factors
influencing the decision to act
Response: Theory of Planned Behavior
Correct answer: Health Belief Model
Score: 0 out of 1

Question 22
These are factors and resources essential to the health of communities and individuals
Response: Social determinants of health
Correct answer: Social determinants of health
Score: 1 out of 1

Question 23
A water tight receptacle which receives the discharge of a plumbing system or part
thereof, and is designed to accomplish the partial removal and digestion of the
suspended solid matter in the sewage through a period detention
Response: Septic tank
Correct answer: Septic tank
Score: 1 out of 1

Question 24
Helping people to gain power over their own lives is defined as
Response: empowerment
Correct answer: empowerment
Score: 1 out of 1

Question 25
The final step in community diagnosis
Response: dissemination and evaluation of results
Correct answer: dissemination and evaluation of results
Score: 1 out of 1

Question 26
The London cholera epidemic of 1848 was traced to the Broad Street pump by
Response: Snow
Correct answer: Snow
Score: 1 out of 1

Question 27
Which health indicator reflects the equity of provision of health care
Response: Health Care Delivery Indicators
Correct answer: Health Care Delivery Indicators
Score: 1 out of 1

Question 28
Determine if what stage (arouse, organize, mobilize)of organizing are the following
situations:

Holding a general assembly regarding the problem of waste disposal in the barangay
Response: Organize
Correct answer: Organize
Score: 1 out of 1

Question 29
The health belief model holds that, before seeking preventive measures, people
generally must believe the following:
Response: The disease at issue is serious, if acquired
Correct answer: The disease at issue is serious, if acquired
Score: 1 out of 1

Question 30
It is the proportion of people with the disease in the screened population who are
identified as ill by the screening test
Response: Sensitivity
Correct answer: Sensitivity
Score: 1 out of 1

Question 31
An individual's ability to read, understand, and use healthcare information to make
effective healthcare decisions and follow instructions for treatment
Response: Health education
Correct answer: Health literacy
Score: 0 out of 1

Question 32
Fetal deaths plus neonatal deaths per 1,000 live births plus fetal deaths
Response: Perinatal mortality rate
Correct answer: Perinatal mortality rate
Score: 1 out of 1

Question 33
Variables used for the assessment of community health
Response: Health indicators
Correct answer: Health indicators
Score: 1 out of 1

Question 34
Which of the following transmissions occurs through contaminated materials, such as
food, clothes, bedding and cooking utensils?
Response: Indirect transmission
Correct answer: Indirect transmission
Score: 1 out of 1

Question 35
Theory on behavior change wherein the focus is on Individuals' motivation and
readiness to change a problem behavior
Response: Precaution Adoption process model
Correct answer: Stages of Change Model
Score: 0 out of 1

Question 36
Which among the factors below in determining the nature of the infection is a measure
of the severity of disease, which can vary from very low to very high?
Response: Virulence
Correct answer: Virulence
Score: 1 out of 1

Question 37
Enumerate the different stages of change in proper order: (1) _____, (2) _____, (3)
_____, (4) _____, (5) _____
Response: [none]
Correct answer: Precontemplation, Contemplation, Preparation, Action, Maintenance
Score: 0 out of 1

Question 38
Susceptibility of a host to a disease can be best altered or controlled through
Response: building the immune system and resistance to agents of diseases
Correct answer: building the immune system and resistance to agents of diseases
Score: 1 out of 1

Question 39
In deciding which families need a more careful assessment on intimacy, the family
physician looks at which parameter?
Response: Affection
Correct answer: Affection
Score: 1 out of 1
Question 40
In Initiation the following should be done except
Response: social investigation
Correct answer: social investigation
Score: 1 out of 1

Question 41
Which of the following is measured by the Family APGAR?
Response: Satisfaction about family relationships
Correct answer: Satisfaction about family relationships
Score: 1 out of 1

Question 42
Dolly is a young mother of 4 adolescents. The younger children help Dolly in their sar-
sari store business when they have no schoolwork. This ability to "fill in" is classified
under which family strength?
Response: Flexibility of family functions and roles
Correct answer: Flexibility of family functions and roles
Score: 1 out of 1

Question 43
Uses of a Family Genogram
Response: Allows the user to analyze hereditary patterns and psychological factors that
punctuate these relationships
Response: Maps out relationships and traits that may otherwise be missed on a
pedigree chart
Response: Quickly identify and understand various patterns in the patient's family
history which may have had an influence on the patient's current state of mind
Response: Graphically represens a family tree that displays detailed data about the
relationships among individuals in a family
Correct answer: Allows the user to analyze hereditary patterns and psychological
factors that punctuate these relationships , Maps out relationships and traits that may
otherwise be missed on a pedigree chart , Quickly identify and understand various
patterns in the patient's family history which may have had an influence on the patient's
current state of mind , Graphically represens a family tree that displays detailed data
about the relationships among individuals in a family
Score: 1 out of 1

Question 44
Intermediate outcomes of addressed social and cultural determinants of health
Response: Social cohesion
Correct answer: Social cohesion
Score: 1 out of 1

Question 45
The following are examples of first order changes of the Family in Later Life EXCEPT:
Response: Helping their children finish school and providing them support while they
look for jobs & become financially stable
Correct answer: Helping their children finish school and providing them support while
they look for jobs & become financially stable
Score: 1 out of 1

Question 46
Determine the level of people's participation:

Health with the people


Response: Community Based
Correct answer: Community Based
Score: 1 out of 1

Question 47
The following are examples of first order changes of the Family in Later Life EXCEPT:
Response: Helping their children finish school and providing them support while they
look for jobs & become financially stable
Correct answer: Helping their children finish school and providing them support while
they look for jobs & become financially stable
Score: 1 out of 1

Question 48
Tito Vhoy refuses to talk with his doctors and even with his best friend Kristeta after
finding out he has HIV
Response: reaction to diagnosis
Correct answer: reaction to diagnosis
Score: 1 out of 1

Question 49
Aims to maintain the status quo, perpetuate the existing health system
Response: Hospital based
Correct answer: Hospital based
Score: 1 out of 1

Question 50
If there are 15,231 total deaths in Country A during calendar year 2012 with a
estimated 2012 mid-year population of 2,010,787. What is the crude death rate?
Response: 757.5 deaths per 100,000 population
Correct answer: 757.5 deaths per 100,000 population
Score: 1 out of 1

Question 51
Engr. De Leon consulted Dr. Sotto due to persistent headache and hypertension. ON
PE, Dr. Sotto found that her blood pressure is at 110/70. In an attempt to determine
whether she is just being anxious, Dr. Sotto asked her: “You seem to be quite anxious
about your blood pressure." Which particular active listening skill is manifested by Dr.
Sotto?
Response: reflecting feeling
Correct answer: reflecting feeling
Score: 1 out of 1

Question 52
Social investigation is done prior to any organization process in the community
Response: True
Correct answer: True
Score: 1 out of 1

Question 53
This is crucial to achieve an equitable health system and setting national priorities in
health care delivery
Response: Strong political will
Correct answer: Strong political will
Score: 1 out of 1

Question 54
Which of the following changes occur when the breadwinner of the family becomes
debilitated?
Response: Second-order change
Correct answer: Second-order change
Score: 1 out of 1

Question 55
Which of the following is not included in the WHO recommendations for performing a
screening test?
Response: Use a screening test that has at least a sensitivity of 85% and a specificity of
80% compared to a gold standard
Correct answer: Use a screening test that has at least a sensitivity of 85% and a
specificity of 80% compared to a gold standard
Score: 1 out of 1

Question 56
Theory on behavior change wherein the focus is on the individuals' attitudes toward a
behavior, perceptions of norms, and beliefs about the ease or difficulty of changing
Response: Stages of Change Model
Correct answer: Theory of Planned Behavior
Score: 0 out of 1

Question 57
In the biopsychosocial model, which among these strategies deal with the SOCIAL
dimension of the patient’s illness?
Response: Probing the patient for possible stressors in his/her life
Correct answer: Conducting a home visit to see the patient’s home situation & local
environment
Score: 0 out of 1

Question 58
Which level of prevention is aimed at reducing the progress or complications of
established disease and is an important aspect of therapeutic and rehabilitation
medicine?
Response: Primary level of prevention
Correct answer: Tertiary level of prevention
Score: 0 out of 1

Question 59
A 16-year-old G1PO comes to your clinic for a routine prenatal examination. Her last
menstrual period was 5 months ago. Physical examination shows diffuse hematomas
on both upper and lower extremities, which the patient tries to hide by wearing a long-
sleeved shirt and long pants. Other Physical Examination findings were unremarkable.
Upon further inquiry, you learn that she comes from a dysfunctional family with an
abusive father and an alcoholic mother. Which of the following interventions would you
prioritize?
Response: Referral to a Women’s Health Desk section to ensure the patient’s safety at
home
Correct answer: Referral to a Women’s Health Desk section to ensure the patient’s
safety at home
Score: 1 out of 1

Question 60
This depends vitally on the empowerment of individuals to challenge and change the
unfair and steeply graded distribution of social resources to which everyone has equal
claims and rights
Response: Policy coherence
Correct answer: Health equity
Score: 0 out of 1

Question 61
Which of the following is not included in the WHO recommendations for performing a
screening test?
Response: Use a screening test that has at least a sensitivity of 85% and a specificity of
80% compared to a gold standard
Correct answer: Use a screening test that has at least a sensitivity of 85% and a
specificity of 80% compared to a gold standard
Score: 1 out of 1
Question 62
The following have negative and detrimental impact on health
Response: Political upheavals
Correct answer: Political upheavals
Score: 1 out of 1

Question 63
The particular aim of ASIN (An Act for Salt Iodization Nationwide) is to eliminate
Response: Iodine deficiency
Correct answer: Iodine deficiency
Score: 1 out of 1

Question 64
Variations in socio-cultural prescriptions are influenced by the following factors:
Response: All of the choices are correct
Correct answer: All of the choices are correct
Score: 1 out of 1

Question 65
Which of the following assessment tools has the disadvantage of being limited to the
assessment of family structure?
Response: genogram
Correct answer: genogram
Score: 1 out of 1

Question 66
Determine if the situation described follows the community organizing principles

A coastal barangay who initiated a mangrove planting activity by themselves


Response: True
Correct answer: True
Score: 1 out of 1

Question 67
Which of the following key attitudes of an effective counselor is realized by the
physician’s ability to accurately and sensitively enter into the inner private world of the
patient and experience at the deepest level what the patient is feeling?
Response: Transparency
Correct answer: Empathy
Score: 0 out of 1

Question 68
Cesar gets married and invites his new wife’s parents into his home. This family is
classified as being:
Response: Extended
Correct answer: Extended
Score: 1 out of 1

Question 69
An age dependency ratio of 105 can be interpreted as
Response: 105 persons will be supported by 100 able bodied persons
Correct answer: 105 persons will be supported by 100 able bodied persons
Score: 1 out of 1

Question 70
The traditional measures of health status widely used because of their ready availability
such as from death certificates which is a legal requirement in many countries
Response: Mortality Rates
Correct answer: Mortality Rates
Score: 1 out of 1

Question 71
Theory on behavior change wherein the focus is on the Individuals' journey from lack of
awareness to action and maintenance
Response: Social Cognitive Theory
Correct answer: Precaution adoption process model
Score: 0 out of 1

Question 72
Determine if what stage (arouse, organize, mobilize)of organizing are the following
situations:

Participating in a fun run supporting the advocacy against domestic violence


Response: Mobilize
Correct answer: Mobilize
Score: 1 out of 1

Question 73
A measure of prognosis that identifies the number of those who died among those
diagnosed with the disease against those who were not diagnosed with the disease
Response: Relative survival
Correct answer: Relative survival
Score: 1 out of 1

Question 74
Restriction or lack (resulting from an impairment) of ability to perform an activity in the
manner or within the range considered normal
for a human being
Response: disability
Correct answer: disability
Score: 1 out of 1

Question 75
Determine if the situation described follows the community organizing principles

A livelihood project chosen by the women of Barangay 189 choosing to make native
delicacies
Response: True
Correct answer: True
Score: 1 out of 1

Question 76
The use of this rate is as a measure of overall health status for a given population
Response: Infant mortality rate
Correct answer: Infant mortality rate
Score: 1 out of 1

Question 77
The highest score minus the lowest score equals the:
Response: Range
Correct answer: Range
Score: 1 out of 1

Question 78
Which of the following statements correctly refers to the biopsychosocial approach to
illness?
Response: It attempts to replace the traditional method of diagnosis and management
Correct answer: In the biopsychosocial approach, the single individual is considered to
be the lowest unit of the social hierarchy
Score: 0 out of 1

Question 79
The natural history of a disease process commences with
Response: interaction of the host and disease agent factors with the environment
Correct answer: interaction of the host and disease agent factors with the environment
Score: 1 out of 1

Question 80
Liberating
Response: Community Managed
Correct answer: Community Managed
Score: 1 out of 1

Question 81
Which measure/s of central tendency is/are very sensitive to or easily affected by
extreme values?
Response: Mean
Correct answer: Mean
Score: 1 out of 1
Question 82
When a researcher wants to test for the association between two categorical variables,
what is the appropriate statistic to use?
Response: T test
Correct answer: Chi square
Score: 0 out of 1

Question 83
Protection of health by personal and communal efforts, such as enhancing nutritional
status, providing immunizations, and eliminating environmental risks are the actions
usually done in what level of prevention?
Response: Primordial level of prevention
Correct answer: Primordial level of prevention
Score: 1 out of 1

Question 84
It is a process by which patients and providers consider outcome probabilities and
patient preferences and reach a mutual decision
Response: Shared decision making
Correct answer: Shared decision making
Score: 1 out of 1

Question 85
Autopsies may be performed on patients who die in accredited hospitals subject to the
following requirements:
Response: All of the choices are correct
Correct answer: All of the choices are correct
Score: 1 out of 1

Question 86
The Milk Code success is anchored upon
Response: Cooperation and support of physicians
Correct answer: All of the choices are correct
Score: 0 out of 1
Question 87
Any loss or abnormality of psychological, physiological or anatomical structure or
function
Response: impairment
Correct answer: impairment
Score: 1 out of 1

Question 88
What scale of measurement is used to compare quantitative indicators involving two
related populations?
Response: Paired T-test
Correct answer: Paired T-test
Score: 1 out of 1

Question 89
Ruth, 39, consults you for joint and muscle pains. Physical examination and laboratory
results are normal. On probing, you note that she prefers consulting her siblings about
her personal problems. Peter, her husband, is an OFW and has not been home for the
past 2 years, but they have had more than adequate financial resources since he left.
Ruth feels that Peter is not supporting her wishes to transfer to a new neighborhood that
is closer to her parents’ house. Ruth is perennially dissatisfied with the way her children
respond to her emotional outbursts. She also complains that she hardly sees her two
teenaged kids, who are always out partying with friends. Ruth’s family can be said to be:
Response: Severely dysfunctional
Correct answer: Severely dysfunctional
Score: 1 out of 1

Question 90
The process of enabling people to increase control over their health and its
determinants, and thereby improve their health
Response: Advocacy
Correct answer: Health Promotion
Score: 0 out of 1

Question 91
The square root of variance:
Response: Standard Deviation
Correct answer: Standard Deviation
Score: 1 out of 1

Question 92
True of a Family Genogram
Response: Must consist of 3 or more generations with each generation identified by
Roman numerals
Response: Family name is placed above each major family unit
Response: Index patient is identified with an arrow
Response: Date must be indicated when it was made to be able to adjust the ages over
time
Correct answer: Must consist of 3 or more generations with each generation identified
by Roman numerals , Family name is placed above each major family unit , Index
patient is identified with an arrow , Date must be indicated when it was made to be able
to adjust the ages over time
Score: 1 out of 1

Question 93
The family map is a tool that facilitates the communication of information about a family
system. The two parallel lines that connect two personalities indicate which of the
following relationships?
Response: functional
Correct answer: functional
Score: 1 out of 1

Question 94
This is computed as the quotient of the standard deviation divided by the mean
Response: Coefficient of variation
Correct answer: Coefficient of variation
Score: 1 out of 1

Question 95
Who among the following is a candidate for hospice care?
Response: A 60-year old with stage 4 pancreatic cancer who is not in pain at the
moment
Correct answer: A 60-year old with stage 4 pancreatic cancer who is not in pain at the
moment
Score: 1 out of 1

Question 96
Rahizza, delivered in a lying-in hospital and was discharged with no complications.
However, she came back with foul-odor discharge and fever. She was subsequently
admitted and eventually died. This is the most likely cause of Rahizza's death?
Response: Infection
Correct answer: Infection
Score: 1 out of 1

Question 97
The crude death rate in Country A is 150/100,000. The crude death rate in a smaller,
developing country is 75/100,000. Based
on these data, which one of the following statements best explains the data?
Response: Crude death rates are usually higher in developed countries because of a
higher proportion of older persons in the population
Correct answer: Crude death rates are usually higher in developed countries because of
a higher proportion of older persons in the population
Score: 1 out of 1

Question 98
Neonatal Mortality Rate
Response: Number of individuals dying at less than 28 days of age per 1,000 live births
Correct answer: Number of individuals dying at less than 28 days of age per 1,000 live
births
Score: 1 out of 1

Question 99
This person is NOT authorized under the Code of Sanitation to perform autopsy and
dissection of remains
Response: Health officers
Correct answer: Any Licensed physician
Score: 0 out of 1
Question 100
Community Diagnosis include the following
Response: All of the choices
Correct answer: All of the choices
Score: 1 out of 1

You might also like